Inner automorphisms (need clarification)

  • Thread starter Thread starter bennyska
  • Start date Start date
Click For Summary
SUMMARY

The discussion focuses on proving that the inner automorphisms of a group G form a normal subgroup of the group of all automorphisms of G under function composition. The user initially established that all automorphisms of G form a group, denoted as A. To complete the proof, the user must demonstrate that the set of inner automorphisms, I, is a normal subgroup of A by utilizing the properties of conjugation and showing that every left coset is also a right coset.

PREREQUISITES
  • Understanding of group theory concepts, specifically automorphisms and normal subgroups.
  • Familiarity with function composition in the context of group operations.
  • Knowledge of conjugation and its role in defining inner automorphisms.
  • Ability to construct mathematical proofs, particularly in group theory.
NEXT STEPS
  • Study the properties of normal subgroups in group theory.
  • Learn about the specific characteristics of inner automorphisms and their relationship to conjugation.
  • Review examples of groups and their automorphisms to solidify understanding.
  • Explore the implications of the normality of subgroups in various mathematical contexts.
USEFUL FOR

Students and mathematicians studying group theory, particularly those interested in automorphisms and subgroup properties. This discussion is beneficial for anyone looking to deepen their understanding of the structure of groups and their automorphisms.

bennyska
Messages
110
Reaction score
0

Homework Statement



Show that the inner automorphisms of a group G form a normal subgroup of the group of all automorphisms of G under function composition.

Homework Equations



in the previous problem, i showed that all automorphisms of a group G form a group under function composition


The Attempt at a Solution



so i need help understanding the question. in the previous problem, assuming i did it correctly, i let A be the set of all automorphisms and showed it was a group in G. in this problem, am i to let say I be the set of all inner automorphisms and show it is a normal subgroup of A?

here is my previous proof.

: Claim: All automorphisms of a group G form a group under function composition.
Proof: Let A be the set of automorphisms of a group G, and let μ(g) and σ(g) be in A, with g in G. Since μ(g) and σ(g) are automorphisms, it follows that the mapping of μ(g)◦σ(g) defined as μ(σ(g)) is closed under function composition.
Let λ(g) be in A. Then
(μ(g)◦σ(g))◦λ(g) = μ(σ(g))◦λ(g)
= μ(g)◦σ(g)◦λ(g)
= μ(g)◦(σ(g)◦λ(g))
so A is associative.
Consider μ:G→G such that μ(g) = g. Then μ is the identity.
Consider σ:G→G such that σ(g) = a. Now consider λ:G→G such that λ(a) = g. Then σ(λ(a)) = σ(g) = a, and λ is the inverse of σ.
Since A is the set of automorphisms of G, it follows that for any σ in A, then σ is homomorphic. Hence, the set of automorphisms of a group G forms a group under function composition.
 
Physics news on Phys.org
You are asked to show that the set of inner automorphisms is a normal subgroup of the group of automorphisms of G.
An inner automorphism is a specific type of automorphism: http://en.wikipedia.org/wiki/Inner_automorphism, so it is natural that it will form a subset. If you've already shown that the set of automorphisms on group G is a group, then you have already showed closure (under function composition).

You are correct in showing the identity element, but you haven't used the specific property of conjugation that is the defining property of your set of inner automorphisms. You must show invertibility to show it is a subgroup and then that every left coset is a right coset, to show it's normal.
 
Question: A clock's minute hand has length 4 and its hour hand has length 3. What is the distance between the tips at the moment when it is increasing most rapidly?(Putnam Exam Question) Answer: Making assumption that both the hands moves at constant angular velocities, the answer is ## \sqrt{7} .## But don't you think this assumption is somewhat doubtful and wrong?

Similar threads

  • · Replies 2 ·
Replies
2
Views
2K
  • · Replies 2 ·
Replies
2
Views
1K
  • · Replies 8 ·
Replies
8
Views
2K
  • · Replies 2 ·
Replies
2
Views
2K
  • · Replies 6 ·
Replies
6
Views
2K
  • · Replies 3 ·
Replies
3
Views
2K
  • · Replies 5 ·
Replies
5
Views
2K
  • · Replies 3 ·
Replies
3
Views
2K
  • · Replies 1 ·
Replies
1
Views
1K
  • · Replies 3 ·
Replies
3
Views
2K